Đến nội dung

ecchi123 nội dung

Có 177 mục bởi ecchi123 (Tìm giới hạn từ 28-04-2020)



Sắp theo                Sắp xếp  

#684956 Tuần 3 tháng 6/2017: Chứng minh rằng $\angle ADO= \angle OAG...

Đã gửi bởi ecchi123 on 19-06-2017 - 07:43 trong Chuyên mục Mỗi tuần một bài toán Hình học

Lời giải  bài 2 :

quangtrong15b.png

 Gọi $(BPC),(QBC)$ cắt $AB$ tại $F,E$ Có $\widehat{FXP}=\widehat{FBP}=\widehat{QBC}=\widehat{QYC}$ nên $ \Delta  AFX \sim \Delta ACY$ . từ đó $\frac{AX}{AY}=\frac{AF}{AC}$

Mặt khác $\widehat{AFP}=\widehat{PCB}=\widehat{QCA}$ nên  $ \Delta  AFP \sim \Delta ACQ$ từ đó $\frac{AP}{AQ}=\frac{AF}{AC}$

 Suy ra dc $PQ$ song song $XY$ , nên $AD$ chia đôi $PQ$

Giờ ta cm $D$ thuộc $BC$

 Gọi $ AP,AQ$ cắt $BC$ tại $M,N$

Ta có $ \widehat{YQC}=\widehat{FPX}    ,     \widehat{FBP}=\widehat{QBC}   ,     \widehat{QBE}=180-\widehat{ABQ}=180-\widehat{PBC}   ,       \widehat{YBA}=\widehat{YBC}+\widehat{ABC}=\widehat{ABC}+\widehat{FPX}=180-\widehat{XPC}$

nên 2 tứ giác $PFXC,QCYE$ đồng dạng với nhau nên $B(PFXC)=B(QCYE) => (PAXM)=(QNYA)=(YAQN)$ . từ đó $MN,PY,QX$ đồng quy




#684951 Tuần 3 tháng 6/2017: Chứng minh rằng $\angle ADO= \angle OAG...

Đã gửi bởi ecchi123 on 18-06-2017 - 21:48 trong Chuyên mục Mỗi tuần một bài toán Hình học

 Lời giải bài 1 :

Diều cần Cm tường đương $OA^2=OG.OD$ . tức là $(O)$ trực giao với $(DLJ)$

 

Ngịch đảo tâm A phương tích bất kì , ta biến thành bài toán sau : Cho tam giác $ABC$. $E,F$ bất kì trên $AC,AB$ . $(AEF),(ABC)$ cắt nhau tại $D$. $J$ là trực tâm tam giác $AEF$ .$AJ$ cắt $ABC$ tại $L$ . Chứng minh  : tâm $(DJL)$ thuộc $BC$

Untitledddddd.png

 

Giải : $EF$ cắt $BC$ tại $P$

 Lấy $G$ đối xứng với $L$ qua $BC$ . dễ thấy $\Delta  GBC \sim  \Delta JFE$

Vị tự tâm $D$ tỉ số $DF/DB$ và xoay góc $(DB,DF)$ biến $B,C  \mapsto F,E$ . nên  $G  \mapsto J$ từ đó

 $ \widehat{GLJ}=\widehat{GLC}-\widehat{JLC}=\widehat{AFE}-\widehat{ABC}=\widehat{BPF}=\widehat{FDB}=\widehat{JDG} $ ($D$ là đ miquel )

nên $(DGJL)$ nội tiếp . có tâm thuộc trung trực $GL$ tức $BC$ , ta có dpcm




#684147 Tuần 2 tháng 6/2017: Chứng minh rằng $UV \perp AD$.

Đã gửi bởi ecchi123 on 11-06-2017 - 23:49 trong Chuyên mục Mỗi tuần một bài toán Hình học

Lời giải bài 1:

Đầu tiên ta có 1 phát biểu (!) : Cho $O(ABCD)=-1$ .  $OA\perp O'A', OB\perp O'B', OC\perp O'C', OD\perp O'D'$ thì $O'(A'B'C'D')=-1$

 

quantrong14.png

 GỌi $(K)$ cắt $AB,AC$ tại $R,S$ . $(ARS)$ cắt $(O)$ tại $G,A$ 

Ta có $AG,EF$ , tiếp tuyến tại $D$ đồng quy tại $Y$ do là TDP của các cặp 3 đường tròn $(O),(K),(ARS)$

 Nếu $AD$ cắt $(K)$ tại $X$ thì $XRDS$ là tứ giác điều ghòa , và  tiếp tuyến tại $X,D$ và $AG,EF$ đồng quy nên $A(GDRS)=-1$

 vậy nên  theo (!)  $O(EFJM)=-1$ . điều đó chứng tỏ $O$ là trung điểm $MN$ 

Ta có $K(MNOY)=-1$ vậy nên theo (!) : $D(PQXY)=-1$ . khi đó $PQDX$ điều hòa , nên $PQ$ đi qua $Y$

Từ đó ta lại có $A(PQXY)=-1$ . Gọi  $MU$ cắt $NV$ tại $T$ . $Z$ sao cho $TZ$ vuông $AD$ .  Dựng $O'$ sao cho $TO'$ vuông $AG$ . Vậy theo (!)  thì $T(ZO'MN)=-1$ nên $TO'$ đi qua $O$ . mà $TO$ vuông $AG$ nên $TO$ đi qua $J$

Tóm lại ta có $T(UVZO)=-1,O(UVMT)=-1$ và $TZ$ song song  $MO$ nên $UV$ song song $OM$ , tức vuông góc với $AD$

 

 




#681550 VMF's Marathon Hình học Olympic

Đã gửi bởi ecchi123 on 22-05-2017 - 21:54 trong Hình học

Bài toán 196. Cho tam giác $ABC$ ngoại tiếp $(I)$ có tiếp điểm với $BC$ là $D$ , đường cao $BE,CF$ cắt nhau tại $H$.  Đường tròn nội tiếp tam giác $AEF$ cắt $EF,FA,AE$ tại $P,R,Q$ . Lấy $K$ sao cho $IK \perp IA$ và $DK=DI$ . $HK$ cắt $AI,RQ$ tại $J,L$ . Lấy $G$ đối xứng với $J$ qua $PL$ . $QR$ cắt $EF$ và đường thẳng qua $J$ song song với $EF$ tại $S,T$. $X,Y$ là trung điểm của $AS,JT$.Chứng minh rằng $LG \perp XY$ 

eeeeeeeeeeee.png




#681456 Tuần 4 tháng 5/2017: Chứng minh rằng $MY \parallel KR$.

Đã gửi bởi ecchi123 on 21-05-2017 - 23:11 trong Chuyên mục Mỗi tuần một bài toán Hình học

Lời giải bài 2

Gọi $Y$ là giao 2 tiếp tuyến tại $B,C$. $Z$ là trung điểm $OY$ . $J$ đối xứng với $O$ qua $BC$ . $N,M$ là trung điểm cung $BC,BC$

Ta có : $ON^2=OM.OY=OJ.OZ$ , suy ra $AZ,AJ$ đối xứng nhau quá phân giác góc $A$

Ta sẽ chứng minh $AX$ đi qua $Z$ cố định

Nghịch đảo $N$ tâm $A$ , phương tích $AB.AC$ hợp đối xứng trục phân giác góc $A$

Biến $H,B,C,O,BC \mapsto P,C,B,T,(O)$ . Trong đó $T$ đối xứng với $A$ qua $BC$

Gọi $BT,CP$ cắt $OC,BH$ tại $K,L$  mà $\widehat{OCA}=\widehat{BTA},\widehat{ABH}=\widehat{OPC}$ nên $KACT,ALPB$ vội tiếp

Từ đó $\widehat{KAC}=180-\widehat{BTC}=180-(180-BPC-\widehat{BAC})=180-LAC$

Suy ra $K,A,L$ thẳng hàng 

Áp dụng định lí Dersagues cho 2 tam giác $BHT,CPO$ , khi đó $TO,HP,BC$ đồng quy tại $X'$

Theo tales thì $AX'$ đi qua $J$

Chính vì vậy phép nghịch đảo $N$ :$X' \mapsto X$

Vậy $AX,AJ$ đẳng giác  , nên $AX$ đi qua $Z$ cố định do $AZ,AJ$ đối xứng nhau quá phân giác góc $A$

bbbbbbbbbb.png

 




#681299 Chứng minh I,J, P, H đồng viên.

Đã gửi bởi ecchi123 on 20-05-2017 - 17:03 trong Hình học

Cho tam giác ABC đường cao AH.I,J theo thứ tự là tâm bàng tiếp góc B,C của các tam giác ABH,ACH. Gọi P là tiếp điểm của BC với đường tròn nội tiếp tam giác ABC. Chứng minh bốn điểm I,J,P,H đồng viên.

Lời giải :

DO các phân giác ngoài , dễ thấy $\widehat{JHI}=90^o$

Ta cần cm  $\widehat{JDI}=90^o$ , Gọi $(I),(J)$ cắt $BC$ tại $E,F$

Ta có $ED=\frac{AB+BC-AC}{2}-BH+JE=\frac{AB+BC-AC}{2}-BH+\frac{AH+AC-HC}{2}=\frac{AH+AB-HC}{2}=IF$

CMTT ta có $FD=JE$ , suy ra 2 tam giác $JED,DFI$ bằng nhau , từ đó có $\widehat{JDI}=90^o$ nên $DHIJ$ nột tiếp

vvvvvvvvvvvvv.png




#680723 Tuần 3 tháng 5/2017: đường thẳng $AQ$ luôn đi qua một điểm cố định...

Đã gửi bởi ecchi123 on 14-05-2017 - 22:25 trong Chuyên mục Mỗi tuần một bài toán Hình học

Lời giải bài 1 :

Gọi $(K)$ tiếp xúc với $(O)$ tại $Y$ . $I$ là tâm nội tam giác $ABC$.

Gọi $G$ là trung điểm cung $BC$ , $AG$ cắt $BC$ tại $T$ , ta có $M(GNAH)=-1$  nên  gọi $Q'$ là giao của $MN$ với $AG$ thì $(Q'GAT)=-1$. ta chứng minh $Q'$ trùng $Q$ , từ đó $AQ$ đy qua $G$ cố định

Thật vậy , tức là ta cần cm $X(PGAT)=-1$ với $X$ là giao của $PJ$ và $BC$

Gọi tiếp tuyến tại $P,Y$ cắt nhau tại $V$ thì xét trục đẳng phương $(K),(O),(PBC)$ thì $V$ thuộc $BC$ 

Suy ra $\frac{YB}{YC}=\sqrt{\frac{VB}{VC}}=\frac{PB}{PC}$ nên phân giác góc $BPC,BYC$ cắt nhau trên $BC$

Mặt khác , theo bài toán protassov thì $PJ$ là phân giác góc $BPC$ , từ đó suy ra $XY$ là phân giác góc $BYC$ 

Gọi $PL$ là phân giác ngoài $BPC$ suy ra $(LXBC)=-1$

$(K)$ cắt $AB,AC$ tại $E,F$ , $YE,YF$ cắt $(O)$ tại $Z,S$

ta so $ZS$ song song $EF$ nên $\frac{ZE}{ZY}=\frac{SY}{SY}=>\frac{ZE}{ZB}.\frac{ZB}{ZY}=\frac{SY}{SC}.\frac{SC}{SY}=>\frac{BE}{FC}=\frac{BY}{YC}=\frac{XB}{XC}$ suy ra $EF$ đy qua $L$ theo menelaus

Ta có $L$ đối cực với $XA$ qua $(K)$ nên $KL$ vuông góc với $XA$ tại $W$ , mặt khác $KP,KY$ là tiếp tuyến của $(XL)$

Chính vì vậy tứ giác $WPLY$ là tứ giác điều hòa  nên $X(WLPY)=-1=>X(PGAT)=-1$ , khi đó $XP$ đy qua $Q'$ , nên $Q$ trùng $Q'$ có dpcm

zxvxcvxcvxcv.png




#680102 CMR:trực tâm tam giác POQ nằm trên AC

Đã gửi bởi ecchi123 on 09-05-2017 - 20:03 trong Hình học

bạn có thể đưa ra lời giải sử dụng phép vị tự quay giúp mình được không?

Ý tưởng vị tưởng quay là lấy thêm điểm $M$ đó bạn




#678553 Tuần 4 tháng 4/2017: Đường tròn pedal của $A$ ứng với tam giác...

Đã gửi bởi ecchi123 on 24-04-2017 - 23:45 trong Chuyên mục Mỗi tuần một bài toán Hình học

Lời giải bài 1 :

Quantrong 9''.png

. Đối trung $AG$của tam giác $ABC$.$MA$ cắt $BL$ tại $A'$ .  $BM$ cắt $CN$ tại $P'$

Ta sẽ chứng minh $AP' \perp  BC$  .Ta có $\widehat{NAB}=\widehat{MAC}$ .và $NBA=MCA=90$  Dựng $U$ nằm ngoài tam giác và $AU \perp BC$ sao cho $\frac{AU}{BC}=\frac{AB}{BN}=\frac{AC}{CM}=>\bigtriangleup NBC\sim \bigtriangleup BAU,\bigtriangleup MBC\sim \bigtriangleup CUA$ Từ đó $BU\perp CN ,CU\perp BM$ nên $P'$ là trực tâm tam giác $UBC$ . Khi đó $AP' \perp BC$.

Gọi $LM$ cắt $AP'$ tại $P_1$.Do $L$ là trung điểm của $A'B$ nên $P_1$ là trung điểm $AP'$. trương tự suy ra $P'=P_1$ và là trung điểm $AP'$

$V_{A}^{2} :K,L,X,Y,Z,P \mapsto  K',L',X',Y',Z',T',P'$ Khi đó $P'$ là giao của $BM,CN$ ta chứng minh $(X'Y'Z')$ tiếp xúc $BC$

 

Ta sẽ chứng minh $AP' \perp  BC$  .Ta có $\widehat{NAB}=\widehat{MAC}$ .  Dựng $U$ nằm ngoài tam giác và $AU \perp BC$ sao cho $\frac{AU}{BC}=\frac{AB}{BN}=\frac{AC}{CM}=>\bigtriangleup NBC\sim \bigtriangleup BAU,\bigtriangleup MBC\sim \bigtriangleup CUA$ Từ đó $BU\perp CN ,CU\perp BM$ nên $P'$ là trực tâm tam giác $UBC$ . Khi đó $AP' \perp BC$

 

Tiếp theo ta có : $(AX'Z'BL')(AX'Y'CK')$ tiếp xúc với $BC$,.Mặt khác , Gọi tiếp tuyến tại $A$ của $(O)$ cắt $BC$ tại $T$ . Khi đó : $MN$ đy qua $T$ . $AO$ cắt $(O)$ tại $J$ .Khi đó , do $(TGBC)=-1$ nên $J,G,P'$ thẳng hàng

 

Nghịch đảo tâm $A$ phương tích bất kì : ( tên điểm vẫn giữ nguyên khi nghịch đảo )

Cho tam giác $AB$  nội tiếp $(Q)$ có đường cao $AJ$ . tiếp tuyến tại $B,C$ của $(ABC)$ cắt nhau tại $X'$. trung tuyến đỉnh $A$ cắt $(ABC)$ tại $G$. $(AGJ)$ cắt $AQ$ tại $P'$ . Đường thẳng qua $P'$ vuông góc với $AQ$ cắt $BX',CX'$ tại $Z',Y'$ . Chứng minh :$(X'Z'Y')$ tiếp xúc $(Q)$

Giải : ,

quantronmg9.2.png

Gọi đường kính $AE$ . $GE$ cắt $BC$ tại $F$ . $R$ là trung điểm $BC$.  

Giả sử : $(BRG)(CRG)$ cắt $BX',CX'$ tại $Z_1,Y_1$ khi đó $\frac{RZ_1}{RY_1}=\frac{GZ_1}{GR}=\frac{GB}{GC}=\frac{AC}{AB}$

Suy ra $\bigtriangleup ABC \sim \bigtriangleup RY_1Z_1 \Rightarrow \widehat{RY_1Z_1}=\widehat{RY_1C}$ nên $R$ là tâm bàng tam giác $X'Y_1Z_1$ . Suy ra $(Q)$ là đường tròn $Mixtilinear$ của tam giác $X'Y_1Z_1$ nên $(X'Y_1Z_1)$ tiếp xúc $(Q)$

 

Ta cần chứng minh $Y_1,Z_1\equiv Y',Z'$ . Gọi $I$ là trung điểm cung $Y_1Z_1$ của $(X'Y_1Z_1)$  , khi đó Ta có các tc sau của dường tròn $Mixtilinear$ được cm ở đây ( https://nguyenvanlin...mixtilinear.pdf )

1) $IG\perp GR$

2) $IG,BC,Y_1Z_1$ đồng quy

 Khi đó $Y_1Z_1$ đy qua $P'$ và $Y_1Z_1$ vuông góc $AQ$( cộng góc ) nên $Y_1,Z_1\equiv Y',Z'$ . Ta có dpcm




#675464 Tuần 4 tháng 3/2017: $PQ$ luôn đi qua một điểm cố định khi $(K...

Đã gửi bởi ecchi123 on 27-03-2017 - 20:37 trong Chuyên mục Mỗi tuần một bài toán Hình học

Lời giải bài 2 :

Đầu tiên Ta có $\widehat{BFP}=\widehat{CEP}=\widehat{BAC}$ nên $FOPB,EOPC$ nội tiếp , Gọi $K,L$ là tâm $(OPB),(OPC)$ , Ta có $(K)=(L)$ . Dựng $X$ sao cho $X$ thuộc $PO$ và $\widehat{XKO}=\widehat{BAC}$ Nên $X$ cố định.  Mặt khác $\widehat{FPE}= \widehat{BAC}$ nên ta có :$\widehat{FKX}=\widehat{ELX}$ Do đó2 tam giác $FKX,ELX$ bằng nhau nên $XE=XF$ suy ra trung trực $FE$ đy qua $X$ cố định

Quangtrong5'.png




#675143 CMR KA là phân giác của góc EKF

Đã gửi bởi ecchi123 on 23-03-2017 - 18:14 trong Hình học

Cho tam giác ABC nội tiếp $\left ( O \right )$. Ba đường cao AD,BE,CF. EF cắt BC ở S. từ S kẻ tiếp tuyến SK tới $\left ( O \right )$. 

a,CMR KA là phân giác của góc EKF 

b,Gọi KD cắt EF ở L. Gọi I là trung điểm BC. AO cắt EF tại M. CMR IL $\perp$ AS và tứ giác BLMC nội tiếp

attachicon.gifVMF_(1).png

a)$K$ của cả 2 trường hợp đều đúng , Mình sẽ chứng minh 1 th , th còn lại làm tương tự :

$K$ cùng phía với $A$ so với $BC$, có $KS^2=SB.SC=SE.SF$ nên $(KEF)$ ts $SK$ . 

Gọi $KF$ cắt $(O)$ tại $Q$ , $CQ$ cắt $FE$ tại $P$ , ta có $\widehat{KCQ}=\widehat{SKQ}=\widehat{KES}$ nên $KECP$ nội tiếp

nên $\widehat{PKC}=\widehat{PEC}=180-\widehat{ABC}=180-\widehat{AKC}$ nên $P,K,A$ thẳng hàng  suy ra $\widehat{PKQ}=\widehat{ACQ}=\widehat{AKE}$ suy ra , $AK$ là phân giác ngoài góc $FKE$

b) Dễ thấy $AM$ vuông góc $FE$ , $AM$ cắt $(O)$ tại $R$ .  GGoij $RH$ cắt $FE$ tại $L'$ . do r $HR$ vuông $Á$ nên ta chỉ cần cm $L$ trùng $L'$

ta có $AM.AR=AD.AH$ nên $DHNR$ nội tiếp suy ra

$\widehat{MDI}=90-\widehat{ADM}=90-\widehat{MRL'}$ nên $L'DIM$ nội tiếp , 

Có $\widehat{MSO}=\widehat{MIO}=90=\widehat{MID}=90-\widehat{SL'D}$ nên $SO$ vuông góc $LD$ , Mà $S$ liên hợp với $D$ qua $(O)$ nên $L'D$ là đường đối cực của $S$ qua $(O)$ nên $L',D,K$ thẳng hàng nên $L$ trùng $L'$ đồng thòi có $LDIM$ nội tiếp , 

( vì bạn sửa lại đề bài nên ko tiện tay sửa lại phần a , th hình như bạn thì làm tương tự cách làm trên , giống hệt luôn )

34.png




#674983 Tuần 3 tháng 3/2017: Bốn điểm $U,V,S,T$ cùng thuộc một đường tròn.

Đã gửi bởi ecchi123 on 21-03-2017 - 21:03 trong Chuyên mục Mỗi tuần một bài toán Hình học

Lời giải bài 2 :  Trước khi giải , ta có 2 bài toán sau :

 

 

 BT1:  Cho tam giác $ABC$ nt $(O)$ , $P,Q$ là 2 điểm liên hợp đẳng giác trong tam giác , $AQ$ cắt $(O)$ tại $G$ . $GP$ cắt $BC$ tại $H$ . Chứng minh : $HQ$ song song $AG$ .

 

trung1.png

Lời giải : Gọi $AP$ cắt $BC, (O)$ tại $K,L$ . $\widehat{KPC}=\widehat{PAC}+\widehat{ACP}=\widehat{QCB}+\widehat{BCG}=\widehat{QCG}$

 

Nên ta có 2 tam giác $PLC,CGQ$ đồng dạng với nhau $\rightarrow LP.QG=CL.CG$ , 

 

Mặt khác 2 tam giác $LKC,GCA$ đồng dạng với nhau  $\rightarrow LK.AG=CL.CG$

 

Chính vì vậy $\frac{AG}{QG}=\frac{LP}{LK}=\frac{GP}{GH}$ nên $HQ$ song song $AG$

 

BT2: Cho Hình bình hành $ABCD$ cố định và 1 điểm $S$ cố định nằm bất kì . 1 đường thẳng $d$ thay đổi qua $S$ cắt $BC,CD$ tại $E,F$ . $DE,BF$ cắt $AB,AD$ tại $P,Q$ . Chứng minh : $(APQ)$ Luôn đi qua 2 điểm cố định khi $d$ thay đổi qua $S$

 

trung 3.png

Lời giải : Goại $SD,SB$ cắt $AB,AD$ tại $N,M$ . $R$ là giao của $(AND)$ và $(ABM)$ khác $A$ . Có $R$ cố định . Ta sẽ chứng minh $(APQ)$ đi quá $R$. Gọi $CD$ cắt $BS$ tại $X$

 

 Áp dụng $Menelaus$ với tam giác $ADN$ cát tuyến $SBM$ Có: $\frac{BN}{BA}.\frac{MA}{MD}=\frac{SN}{SD}=\frac{SB}{SX}$

 

Tiếp tục  Áp dụng $Menelaus$ với tam giác $BCX$ cát tuyến $SEF$ Có :$\frac{SB}{SX}=\frac{EB}{EC}.\frac{FC}{FX}=\frac{PB}{AB}.\frac{FC}{FX}$

 

Mặt khác theo $Tales$ Có :$\frac{AM}{MQ}=\frac{AM}{AD}.\frac{AD}{AQ}.\frac{QA}{MQ}=\frac{CD}{CX}.\frac{CF}{CD}.\frac{AQ}{DQ-DM}=\frac{CD}{CX}.\frac{CF}{CD}.\frac{CX}{DF-DX}=\frac{CF}{FX}$

 

Từ 3 điều trên suy ra :$\frac{BN}{BP}=\frac{DM}{MQ}$ . Mặt khác có 2 tam giác $RBN,RDM$ đồng dạng với nhau và có tỉ số trên suy ra 2 tam giác $RBP,RMQ$ đòng dạng nên $\widehat{RPA}=\widehat{RQA}$ Suy ra $APQR$ nội tiếp (dpcm)

 

Trở lại bài toán đầu : 

 

trung 2.png

Bài toán đưa về các đường tròn ngoại tiếp tam giác $AB_iC_i$ thỏa mãn 2 tính chất trên đy qua 1 điểm cố đinh khác $A$ . 

 

Gọi  $AP,AQ$ cắt $(AB_iC_i)$ tại $L_i,T_i$ . $PT_i,QL_i$ cắt $B_i,C_i$ tại $X_i,Y_i$ thì theo BT1  thì $PY_i,QX_i$ song song với $AQ,AP$ .

 

Do $B_i,C_i$ thay đổi theo 2 điều kiện Và với mỗi $Y_i,X_i$ có đặc điểm như trên  thì ta dựng được duy nhất $(AB_iC_i)$ theo cách như trên ( hình vẽ )

 

. Chính vì vậy , ĐỀ bài sẽ đưa về chứng minh $(AL_iT_i)$ đy qua điểm cố định , Đây chính là BT2

 

Ta có dpcm,




#674893 Tuần 3 tháng 3/2017: Bốn điểm $U,V,S,T$ cùng thuộc một đường tròn.

Đã gửi bởi ecchi123 on 20-03-2017 - 21:10 trong Chuyên mục Mỗi tuần một bài toán Hình học

Lời giải Bài 1 :

Dễ thấy $AT=2NQ,AS=2PM$ và $\widehat{NHP}=\widehat{AEN}=\widehat{AFM}=\widehat{QHM}=>\widehat{NHQ}=\widehat{MHP}$

Và ta cũng có 2 tam giác $AFC,AEB$ đồng dạng với nhau nên $\widehat{NEB}=\widehat{MFC}$

Ta có hàng loạt các biến đổi sau để có dpcm:

$\frac{AT}{AS}=\frac{NQ}{MP}=\frac{NQ}{Sin\widehat{NHQ}}.\frac{Sin\widehat{MHP}}{MP}=\frac{HN}{Sin\widehat{NQH}}.\frac{Sin\widehat{MPH}}{HM}=\frac{NH.Sin\widehat{ENH}}{MH.Sin\widehat{FMH}}=\frac{EH.Sin\widehat{NEB}}{FH.Sin\widehat{MFC}}=\frac{EH}{FH}=\frac{CE}{BF}=\frac{AU}{AV}$

Suy ra $STUV$ nội tiếp

Quangtrong4.png




#674305 Chứng minh OA vuông góc với EF

Đã gửi bởi ecchi123 on 15-03-2017 - 06:04 trong Hình học

Cho 2 đường tròn (O-1) và (O-2) cắt nhau tại A và B, M thuộc tia đối tia AB. Qua B kẻ đường thẳng cắt (O-1) và (O-2) tại C,D. MC,MD lần lượt cắt (O-1),(O-2) tại P,Q. Gọi E,F lần lượt là giao của AC với BP; AD với BQ. Gọi (O) là tâm đường tròn ngoại tiếp ACD, chứng minh OA vuông góc với EF. 

:D  :D

https://diendantoanh...70094-cmr-afde/

Lời giải bạn có thể xem ở đây




#673432 VMF's Marathon Hình học Olympic

Đã gửi bởi ecchi123 on 04-03-2017 - 15:56 trong Hình học

Bài toán 186: Cho tam giác $ABC, $đường cao $AD,BE,CF$ đồng quy tại $H.HK$ vuông góc với $EF.M$ thuộc $BE$ sao cho $KM$ song song với $BC.J$ là tâm $(MEF).$ Chứng minh $J \in DK.$

666666.png




#673431 VMF's Marathon Hình học Olympic

Đã gửi bởi ecchi123 on 04-03-2017 - 15:38 trong Hình học

Lời giải bài 185 :  

 

Ta sẽ chứng minh  $K$ thuộc đường thẳng qua $B$ vuông góc với $OO'$

 

 

Để dễ nhìn, ta viết lại đề như sau : Cho tam giác $ABC$ ngoại tiếp $(I),(I)$ tiếp xúc với $BC,CA,AB$ tại $D,E,F.G$ đối xứng với $D$ qua trung điểm $BC.H$ bất kì sao cho $GH$ vuông góc $BC.$ Đường thẳng qua $D$  vuông góc với $HB,HC$ cắt $(I)$ tại $K,J.JK$ cắt $FE$ tại $L.$ Chứng minh $DL$ vuông góc với $IH.$

 

 

Giải : Hạ $DM,DN$ vuông góc với $JK,EF$ , Ta có : $\frac{JM}{KM}=\frac{JM}{MD}.\frac{MD}{MK}=\frac{GH}{BG}.\frac{CG}{HG}=\frac{BG}{CG}=\frac{CD}{BD}=\frac{EN}{FN}$

 

Gọi $FJ$ cắt $KE$ tại $Q,W$ là điểm $Miquel$ của tứ giác toàn phần $JKEF.LQ$ thì theo tỉ số trên có $(LJF),(LMN),(LEK)$ đồng trục $LW.$

Gọi $DQ$ cắt $(I)$ tại $X,R$ đối xứng với $D$ qua $I.$ Do $\overline{QW}.\overline{QL}=\overline{QJ}.\overline{QF}$ nên $Q$ thuộc trục đẳng phương của $(DMNL)$ nên $X$ thuộc $(MNDL),$ mà $DW$ và $IW$ cùng vuông góc với $QL$ (do $QX.QD=QW.QL$ và góc $DXL$ vuông) nên $DR$ vuông góc với $QL \Rightarrow R$ là trực tâm tam giác $QDL$ nên $QR$ vuông góc với $DL.$

 

Bây giờ ta sẽ chứng minh $QR$ song song $HI.$ Dựng $Y$ nằm ngoài tam giác $ABC$ sao cho$\widehat{HBY}=\widehat{\frac{B}{2}},\widehat{HCY}=\widehat{\frac{C}{2}}.$

Ta có tam giác $RJK$ đồng dạng với $HBC$ và theo cách dựng $Y$ ta cũng có được tam giác $QJK$ đồng dạng $YBC.$

Mặt khác , xét tam giác $BCH$ có $BY,BI$ đẳng giác góc $B;CY,CI$ đẳng giác góc $C$ nên $Y,I$ liên hợp đẳng giác tam giác $HBC$ nên $HI,HY$ đẳng giác góc $H.$

Chính vì vậy $\widehat{QRK}=\widehat{YHC}=180^0-\widehat{BHI}$ và  $BH,RK$ song song với nhau nên $QR,HI$ song song với nhau.

Kết hợp với $QR$ vuông góc với $DL$ suy ra $DL$ vuông góc với $IH,$ đpcm.

 

33;;;;;.png



#673130 VMF's Marathon Hình học Olympic

Đã gửi bởi ecchi123 on 01-03-2017 - 18:21 trong Hình học

Bài toán 184. Cho tam giác $ABC$ , $M$ nằm trên đoạn $BC$ , $K,L$ là tâm ngoại tiếp $(ABM),(ACM).D,E,F,X,Y,Z,T$ lần lượt là trung điểm $MA,MB,MC,KB,KM,LM,LC.YZ$ cắt $XT$ tại $N.NL,NK$ cắt $BC$ tại $P,Q.$ Chứng minh $(DPF)$ tiếp xúc $(DEQ).$

32.png




#673127 VMF's Marathon Hình học Olympic

Đã gửi bởi ecchi123 on 01-03-2017 - 18:00 trong Hình học

Bài toán 182. Cho tam giác $ABC$ nhọn, nội tiếp $(O)$ và ngoại tiếp $(I).P$ là điểm bất kì trên cung nhỏ $BC.$

$M,N$ là hai điểm nằm trên cạnh $BC$ sao cho tam giác $MNP$ nhận $(I)$ làm đường tròn bàng tiếp góc $P$ và $(MNP)$ cắt lại $(O)$ ở $X.$

Chứng minh $AX$ đi qua tâm vị tự của $(O),(I).$

Lời giải bài 182 : Đầu tiên ta đi chứng minh $X$ là tiếp điểm của $A-Mix$ với $(O)$

Thật vậy : Gọi $PM,PN$ cắt $(O)$ tại $F,E$ , $EF$ cắt $BC$ tại $Q$ Khi đó $Q$ là điểm $Miquel$ của tứ giác toàn phần $EFMN.PQ$

Ngịch đảo  tâm $X$ phương tích $XM.XE$ hợp đối xứng phân giác góc $MXE$ do $EFMN$ ngoại tiếp nên $XI$ là phân giác góc $QXP$ , (  https://nguyenvanlin...ao-doi-xung.pdf )

có$\widehat{QXB}=\widehat{XBC}-\widehat{XQC}=\widehat{XBC}-\widehat{XFP}=\widehat{PXC}$ nên $XI$ cũng là phân giác góc $CXB$ nên $X$ là tiếp điểm của $A-Mix$ với $(O)$

Xét các tâm vị tự ngoài của $(O),(I),(A-Mix)$ thì theo định lý $Monge$ thì $AX$ đi qua tâm vị tự của $(O),(I).$

23.png




#673083 VMF's Marathon Hình học Olympic

Đã gửi bởi ecchi123 on 01-03-2017 - 12:18 trong Hình học

Bài toán 180. Cho ngũ giác $ABCDE$ lồi, điểm $F$ trên cạnh $AE$ thỏa mãn $\Delta ABC \sim \Delta CDE \sim \Delta BFD.$ Chứng minh $\frac{AF}{FE}=\frac{BF^2}{FD^2}.$

Lời giải bài toán 180. Gọi $DF$ cắt $AC$ tại $G,BF$ cắt $CD$ tại $H,$ có $BCDG,BCDH$ nội tiếp nên $G,B,C,D,H$ cùng thuộc 1 đường tròn.

Theo định lý Pascal thì $BA$ cắt $DE$ tại $I$ thuộc $(GBCDH).$

Do đó $\frac{AF}{FE}=\frac{AB.\sin \widehat{ABF}}{HE.\sin\widehat{FHE}}=\frac{AB.HI}{HE.BC}=\frac{BF^2}{DF^2},$ đpcm.

 

30.png




#672842 VMF's Marathon Hình học Olympic

Đã gửi bởi ecchi123 on 26-02-2017 - 14:01 trong Hình học

Bài toán 178. Cho tam giác $ABC$ nội tiếp $(O)$ có trực tâm $H,$ phân giác $AD.M$ là điểm chính giữa cung $BHC$ của $(BHC).BM,CM$ cắt $AC,AB$ tại $E,F.EF,AM$ cắt nhau tại $P.K$ là tâm $(AEF).$ Chứng minh :$P$ là trực tâm tam giác $KBC$ và $KA=OD.$ 

27.png




#672841 VMF's Marathon Hình học Olympic

Đã gửi bởi ecchi123 on 26-02-2017 - 13:52 trong Hình học

Lời giải bài toán 177.

Bổ đề. Cho tam giác $ABC$ , $D,E$ nằm trên $AB,AC$ , $K,H$ là trực tâm tam giác $ABC,ADE$ , $M,N,P,Q,F$ là trung điểm $BC,CE,DE,BC,DC.$ Khi đó tam giác $AHK$ đồng dạng tam giác $FNM.$

29.png

Chứng minh. Gọi $J,I$ là tâm ngoại $ABC,ADE$ , ta có $\frac{AH}{AK}=\frac{IP}{JQ}=\frac{DE}{BC}=\frac{FN}{FM}$

Và dễ dàng chứng minh :$\widehat{HAK}=\widehat{NFM}$ suy ra $AHK$ đồng dạng tam giác $FNM$

Từ đây còn suy ra $\frac{HK}{MN}=\frac{HA}{FN}=\frac{2PI}{PE}=2tan\widehat{DEI}=2cot\widehat{DAE}$

Trở lại bải toán :

28.png

Gọi $H,G,I,J$ là trực tâm tam giác $BAQ,PAD,PBC,CDQ$ , ta sẽ chứng minh $GJ=HI$

Thật vậy , gọi $M,N$ là trung điểm $AC,PQ$ , khi đó theo Bổ đề $\frac{GJ}{MN}=2 cot\widehat{ADP}$

 Mặt khác , ta cũng có $\frac{HI}{MN}=2cot\widehat{ABC}$ từ đó suy ra$GJ=HI$




#672794 CMR AF//DE

Đã gửi bởi ecchi123 on 26-02-2017 - 00:12 trong Hình học

Mình đã giải bài toán tổng quát của bổ đề này:

Cho tam giác $ABC$ nhọn. Lấy $D$ 1 điểm bất trên đoạn $BC$ không trùng $B,C$. Lấy $E$ 1 điểm trên đoạn $AD$ ($E$ không trùng $A,D$). Gọi $(DEB)$ cắt $AB$ tại $F$ khác $B$ gọi $(DEC)$ cắt $AC$ tại $G$ khác $C$. $EC$ cắt $GD$ tại $I$ $FD$ cắt $BE$ tại $H$. Gọi $J$ tâm đường tròn ngoại tiếp tam giác $EBC$. Chứng minh rằng: $AJ$ vuông góc $HI$.

Cảm ơn bạn đã  có một bài toán tổng quát , Mình có 1 cách giải cho bài toán này

26.png

 ta có $\overline{HF}.\overline{HD}=\overline{HE}.\overline{HB}$ và tương tự với $I$ , ta được $HI$ là trục đẳng phương của $(BEC),(DFG)$ . $K$ là tâm $(DFG)$ thì $JK$ vuông góc $HI$

Gọi $(DFG)$ cắt $AB,AC,AD$ tại $M,N,P$ , dễ thấy $MN$ song song với $BC$ , $\widehat{PNA}=\widehat{ADG}=\widehat{ACE}$ suy ra $PN$ song song $EC$ , tương tự thì $PM$ song song $EB$ , suy ra có phép vị tự tâm $A$ biến tam giác $PMN$ thành $EBC$ nên $K,J,A$ thẳng hàng , Nên $AJ$ vuông góc với $IH$




#672764 VMF's Marathon Hình học Olympic

Đã gửi bởi ecchi123 on 25-02-2017 - 21:01 trong Hình học

Bài toán 176. Cho tam giác $ABC$ nội tiếp $(O),B,C$ cố định, $A$ di chuyển trên cung lớn $BC,$ phân giác $AD,K$ bất kì cố định trên trung trực $BC,$ đường thẳng qua $D$ vuông góc $BC$ cắt $AK$ tại $T.$ Chứng minh $(T,TD)$ luôn tiếp xúc với 1 đường tròn cố định khi $A$ thay đổi.




#672760 VMF's Marathon Hình học Olympic

Đã gửi bởi ecchi123 on 25-02-2017 - 20:49 trong Hình học

Lời giải bài toán 175.

Vị tự tâm $H$ tí số $1/2$ , Gọi $HE,HF$ vuông góc với $AM,AN$ , ta chứng minh $(DEF)$ tiếp xúc với $(Euler)$ của tam giác $ABC$

Trước tiên , $AH$ cắt $BC$ tại $K$ thì$\overline{AH}.\overline{AK}=\overline{AF}.\overline{AD}=\overline{AE}.\overline{AM}$ nên $(DEF)$ đy qua $M$

 Gọi tiếp tuyến tại $A$ cắt $BC$ tại $T$ , theo bài toán quen thuộc thì $E$ thuộc $(A-Apolo)$ Suy ra $TE=TA$ và $ED$ là đối trung của $EBC$ suy ra $ED,EM$ đẳng giác trong  góc $BEC$ nên $\widehat{DEM}=\widehat{BEC}-2\widehat{MEC}=\widehat{B}-\widehat{C}=180^0-\widehat{AOM}$

 Suy ra $\widehat{EDC}=\widehat{AMC}-\widehat{DEM}=90^0-\widehat{MAO}=90^0-\widehat{KAD}=\widehat{ADK}$

Chính vì vậy $ME=MF$ ta có , trung trực của $EF$  vừa đy qua tâm $(Euler)$ của tam giác $ABC$ vừa đy qua tâm $DFE$ và $M$ nên $(DEF)$ tiếp xúc với $(Euler)$ của tam giác $ABC$ tại $M.$

25.png




#672715 CMR AF//DE

Đã gửi bởi ecchi123 on 25-02-2017 - 11:41 trong Hình học

Bạn chứng minh bổ đề trên được không?

Mình xin trích lại bổ đề : , Cho tam giác $ABC$ nt $(O)$ phân giác $BD,CE$ cắt nhau tại $I$ , $J_a,J_b,J_c$ là các tâm bàng , Chứng minh : $OJ_a$ vuông góc với $DE$

Giải : Đàu tiên , có $(O)$ là $(Euler)$ của tam giá c$J_aJ_bJ_c$ 

Dễ thầy $J$ là trực tâm tam giác $J_aJ_bJ_c$ . Gọi $K$ là tâm $(J_{b}J_{c}I)$ Khi đó $O,J_a,K$ thẳng hàng

Xét trục đẳng phương của $(K),(O)$ có $\overline{EB}.\overline{EA}=\overline{EI}.\overline{EI_c}$ và $\overline{DC}.\overline{DA}=\overline{DI}.\overline{DI_b}$ nên $DE$ là trục đẳng phương của $(K),(O)$ khi đó $DE$ vuông góc với $\overline{J_aOK}$

ko quan trọng.png